1
$\begingroup$

Assume that I have the expression for a Christoffel symbol:

$$ \Gamma^\mu_{\alpha \beta}=\frac{1}{2}g^{\mu \lambda}(\partial_\alpha g_{\beta \lambda}+\partial_\beta g_{\alpha \lambda} - \partial_\lambda g_{\alpha \beta}).\tag{1}$$

If the metric $g_{\mu\nu}$ is diagonal then the identity $$g^{\mu\lambda}g_{\lambda\nu}=\delta^\mu_\nu\tag{2}$$ simplifies to the expression $$g^{\mu\mu}g_{\mu\mu}=1.\tag{3}$$

Therefore is the following expression for the Christoffel symbol notationally correct?

$$\Gamma^\mu_{\alpha \beta}=\frac{1}{2g_{\mu \mu}}(\partial_\alpha g_{\beta \mu}+\partial_\beta g_{\alpha \mu} - \partial_\mu g_{\alpha \beta}).\tag{4}$$

Does it obey the Einstein summation convention correctly as the repeated $\mu$ index is not summed over?

If the expression is not correct how should it be written?

Addition

Ok I see the correct manipulation to get an all covariant form using Einstein notation:

\begin{eqnarray}\tag{5} \Gamma_{\gamma\alpha\beta}&=&g_{\gamma\mu}\Gamma^\mu_{\alpha\beta}\\ &=&\frac{1}{2}g_{\gamma\mu}g^{\mu\lambda}(\partial_\alpha g_{\beta\lambda} + \partial_\beta g_{\alpha \lambda} - \partial_\lambda g_{\alpha\beta})\\ &=&\frac{1}{2}\delta^\lambda_\gamma(\partial_\alpha g_{\beta\lambda} + \partial_\beta g_{\alpha \lambda} - \partial_\lambda g_{\alpha\beta})\\ &=&\frac{1}{2}(\partial_\alpha g_{\beta\gamma} + \partial_\beta g_{\alpha \gamma} - \partial_\gamma g_{\alpha\beta}) \end{eqnarray}

That's correct isn't it?

$\endgroup$
0

1 Answer 1

6
$\begingroup$

Your last expression is not valid, for two reasons: first, any given index can only occur twice per term in the Einstein convention, once as an upper index and once as a lower index. Remember that when an index is repeated, it means you sum over it with the metric: $$T^a T_a = \sum_{a,b} g_{ab} T^a T^b$$ You have terms like $\frac{1}{2g_{\mu\mu}}\partial_\alpha g_{\beta\mu}$ which contain $\mu$ three times. That's meaningless in Einstein notation.

The other reason is that the metric doesn't necessarily commute with the partial derivative: $g\partial \neq \partial g$. So you can't convert a factor like $g^{\mu\lambda} \partial_{\alpha}g_{\beta \lambda}$ into $\partial_{\alpha}g^{\mu\lambda} g_{\beta\lambda}$, which I think is what you had in mind there. (You could put in an extra term to account for the commutator if you wanted to: $g\partial = \partial g + [g,\partial]$.)

By the way, it's not true that contracting the result of $g^{\mu\lambda}g_{\lambda\nu}$ gives you $1$. You get $g^{\mu\lambda}g_{\lambda\mu} = \delta^{\mu}_{\ \mu} = \sum_{\mu} 1$, which is the dimensionality of the space. In normal 3+1D space, this gives 4.

$\endgroup$
4
  • 1
    $\begingroup$ I'd be interested to know what someone thinks is wrong with this answer.... $\endgroup$
    – David Z
    Nov 1, 2015 at 13:50
  • $\begingroup$ Thanks for your very helpful comments but I still think $g^{\mu\lambda}g_{\lambda\nu}=\delta^\mu_\nu$ is correct (source Sean Carroll's grtiny document). $\endgroup$ Nov 1, 2015 at 14:03
  • 1
    $\begingroup$ @JohnEastmond He never said $g^{\mu\lambda}g_{\lambda\nu}=\delta^\mu_\nu$ wasn't correct... $\endgroup$
    – jinawee
    Nov 1, 2015 at 14:11
  • $\begingroup$ ok - I see - sorry I was wrong! $\endgroup$ Nov 1, 2015 at 14:16

Your Answer

By clicking “Post Your Answer”, you agree to our terms of service and acknowledge you have read our privacy policy.

Not the answer you're looking for? Browse other questions tagged or ask your own question.